Inscription / Connexion Nouveau Sujet
Niveau école ingénieur
Partager :

Mecanique

Posté par
marko
30-12-12 à 10:33

Une particule de masse m ayant une charge -q négative se déplace dans un condensateur plan, parallèlement à un axe normal à ses deux armatures. L'armature de gauche est portée au potentiel V(o) = 0 et l'armature de droite, distante de L de la précédente armature est portée au potentiel V(L)= V

A l'instant initial t = 0 la particule est en x=0 avec une vitesse nulle.

1) Exprimer la force électrique F s'exerçant sur la particule en fonction de q , V et L.

  F = qV/L

2) Ecrire la relation fondamentale de la dynamique
  
  F  = ma

3) Exprimer la valeur de la vitesse acquise lorsque la particule atteint la position x = L

  a( qv/mL ; 0)  v ( qvt/mL ; 0) OM ( qvt²/2mL ; 0 )

D'ou a x = L on a  t = racine de (2mL²/qv)

d'ou v = racine de (2qv/m)


4) Quel est le travail de la force électrique le long de ce parcours.

W = l'intégrale de f.dr
W = l'intégrale de f. v .dt
W = l'intégrale de (qv)/l * racine de (2qv/m).t
W = (qv)/l * racine de (2qv/m)* l'intégrale de 1.dt
W = (qv)/l * racine de (2qv/m) * [t] entre L et 0
W = qv * racine de (2qv/m).


Mes réponses sont-elles correctes ?

Merci pour votre aide !

Edit Coll : niveau modifié

Posté par
J-P
re : Mecanique 30-12-12 à 10:59

Attention qu'en cours de calcul tu utilised le v en lettre minuscule aussi bien pour la vitesse que pour le potentiel .

3)

qV/L = m.a
a = qV/(mL)

L = at²/2
L = qV/(2mL).t²
t² = 2mL²/(qV)

t = L * racine(2m/(qV))

v = at
v = qV/(mL) * L * racine(2m/(qV))
v = racine(2qV/m)

Avec v en minuscule le vitesse de la particule en position x=L et V en majuscule le potentiel de l'armature de droite.
Axe des déplacements choisi positif de gauche à droite.
-----
4)

W = S(de0àL) F dx = S(de0àL) qV/L dx = qV
-----
Sauf distraction.  

Posté par
marko
re : Mecanique 30-12-12 à 11:43

D'accord merci pour ta réponse !

Pour la question 2) je viens de voir mon cours où il y a écrit que la force électrique = (1/4pi(Eo))* ( q1q2/(r(1,2))²) avec Eo = 8.854.10^-12

Je me suis trompé ou non ?

Pour la question 4), pourquoi on ne prend pas en compte la vitesse à x =  L ?

Posté par
J-P
re : Mecanique 30-12-12 à 14:11

F = (1/4pi(Eo))* ( q1q2/(r(1,2))²) est la force électrostatique entre 2 particules l'une portant une charge q1 et l'autre portant une charge q2, les 2 particules étant distantes de r(1,2)

Ici, ici s'agit de la force exercée sur une particule chargée (charge q) placée dans un champ électrique E
on a \vec{F} = q.\vec{E}
-----
Et le travail d'une force F le long d'un parcours C est W = \int_C \vec{F} \vec{du} avec \vec{du} un déplacement élementaire le long du parcours C.

... ceci ne concerne pas la vitesse.

On peut parfois introduire, dans ce calcul, la vitesse en considérant que \vec{du} = \vec{v} dt

Mais évidemment, il faut éviter la grosse erreur d'intégrer avec la variable t (temps) en utilisant des bornes de distances pour l'intégrale !!!

Et dans le cas présent, l'intégrale W = \int_C \vec{F} \vec{du} est évidente.
-----

Posté par
marko
re : Mecanique 30-12-12 à 14:24

On a donc W = l'intégrale de F.v.dt  où v correspond à la vitesse instantanée  ?

Quand tu dis qu'il ne faut pas intégrer avec la variable t en utilisant des bornes de distances pour l'intégrale, on fait comment alors ?

On a W =  (qv)/l * [t] entre L et 0
     W = qvl/l
     W = qv  

Est ce correct ?

Posté par
J-P
re : Mecanique 30-12-12 à 20:02

Comment on fait ?

Et bien comme ce que je l'ai présenté dans mon message précédent, on n'a aucun besoin de passer par la vitesse dans le cas de l'exercice.

... Et on trouve W = qV comme je l'ai montré.
--------------
Si on veut, à tout prix passer par la vitesse (ce qui devrait être sanctionné par le prof pour détour strictement inutile) :

W = S(de 0 à T) F.v.dt et on sait que v = a.t avec a = qV/(mL) (voir mon premier message).

avec T l'instant d'arrivée de la charge sur la plaque de droite.

Donc, comme montré dans mon 1er message : T =  L * racine(2m/(qV))

On a donc W = S(de 0 à T) F.v.dt = S(de 0 à T) F.a.t.dt

W = S(de 0 à T) qV/L  * qV/(mL) * t dt

W = q²V²/(2mL²) . [t²](de 0 à T)

W = q²V²/(2mL²) * T²

W = q²V²/(2mL²) (L * racine(2m/(qV)))²

W = q²V²/(2mL²) (L² * (2m/(qV))

W = qV (ATTENTION, V est la différence de potentiel entre les 2 plaques ... et pas la vitesse).

Inutile de dire que celui qui passe par ce chemin montre qu'il ne comprend pas ce qu'il fait, puisqu'on arrive à la solution directement par :

W = S(de0àL) F dx = S(de0àL) qV/L dx = qV
-----

Attention, car écrire ... [t](entre L et 0) est extrêmement faux.

On ne peut pas donner à un temps des mesures de longueurs ... et c'est pourtant ce que tu fais en écrivant ... [t](entre L et 0)




Mentions légales - Retrouvez cette page sur l'île de la physique - chimie
© digiSchool 2024

Vous devez être membre accéder à ce service...

Pas encore inscrit ?

1 compte par personne, multi-compte interdit !

Ou identifiez-vous :


Rester sur la page

Désolé, votre version d'Internet Explorer est plus que périmée ! Merci de le mettre à jour ou de télécharger Firefox ou Google Chrome pour utiliser le site. Votre ordinateur vous remerciera !